0 Daumen
371 Aufrufe

Aufgabe:

Seien \(a_0 = \frac{1}{2} \) und \(a_{n+1} = (2 − a_n)a_n \), fuer n ≥ 0.
(a) Zeigen Sie, dass 0 < a_n < 1 fur alle n ∈ N gilt.

Avatar von

3 Antworten

0 Daumen
 
Beste Antwort

Aloha :)

Wir betrachten die rekursiv definierte Folge$$a_{n+1}=(2-a_n)a_n\quad;\quad a_0=\frac12$$

Wir zeigen die Behauptung \(0<a_n<1\) durch vollständige Induktion. Wegen \(0<a_0=\frac12<1\) ist die Induktionsverankerung klar. Zum Induktionsschritt von \(n\) auf \((n+1)\) stellen wir den Folgenterm etwas um:$$a_{n+1}=-a_n^2+2a_n=(-a_n^2+2a_n\;\underbrace{-1)+1}_{=0}=-(a_n^2-2a_n+1)+1=1-(a_n-1)^2$$und gehen von der Induktionsvoraussetzung aus:$$0<a_n<1\implies -1<a_n-1<0\implies0<|a_n-1|<1\implies 0<(a_n-1)^2<1$$$$\implies0>-(a_n-1)^2>-1\implies 1>1-(a_n-1)^2>0\implies 1>a_{n+1}>0\quad\checkmark$$

Avatar von 148 k 🚀

danke, alles ist klar ausser einer gleichung, wie hast du die gleichung umgeformt ?

\((-a_n^2+2a_n\;\underbrace{-1)+1}_{=0}=-(a_n^2-2a_n+1)^2+1=1-(a_n-1)^2\)?

es ist einfach wenn ich \(1-(a_n-1)^2\)  finde, die ganze Induktion zu schreiben.

aber wie soll ich wissen, wann ich -1+1 schreiben soll ? und warum hast du \(-(a_n^2-2a_n+1)^2\) quadriert? oder geht es immer so bei Folgen, oder gibt es einen regel? ich finde es ist nicht sehr einfach und man soll am zuerst mehrere umformungen versuchen, um eine loesung zu finden..

Ausgehend von$$a_{n+1}=-a_n^2+2a_n$$habe ich zuerst eine "nahrhafte Null" addiert:$$a_{n+1}=-a_n^2+2a_n\,\underbrace{-1+1}_{=0}$$Dann habe ich die Summanden neu geklammert:$$a_{n+1}=(-a_n^2+2a_n-1)+1$$das Minuszeichen aus der Klammer gezogen:$$a_{n+1}=-(a_n^2-2a_n+1)+1$$um schließlich die 2-te binomische Formel zu bemühen:$$a_{n+1}=-(a_n-1)^2+1$$was dasselbe ist wie:$$a_{n+1}=1-(a_n-1)^2$$

+1 Daumen

Tipp: Es ist \(a_{n+1}=1-(1-a_n)^2\).

Avatar von 3,5 k
Hast du einen Tipp, wann man diese Art der Anwendung der binomischen Formel jeweils in Betracht ziehen sollte?
0 Daumen

Es gilt sogar \( \frac{1}{2} \le a_n \le 1  \)

Bew. durch vollst. Induktion:

Für n=0 ist es gegeben.

Und es ist \(  a_{n+1}= 2a_n - a_n^2  \)

Wenn  \( \frac{1}{2} \le a_n \le 1  \) für ein n gilt , dann folgt

\( 1 \le 2a_n \le 2  \)   und   \( \frac{1}{4} \le a_n^2  \le 1  \)

Subtrahieren gibt

\( \frac{3}{4} \le 2a_n - a_n^2  \le 1  \)

also \( \frac{3}{4} \le  a_{n+1} \le 1  \), also auch

\( \frac{1}{2} \le a_{n+1} \le 1  \). q.e.d.

Und damit nat. auch \( 0 \le a_{n} \le 1  \) für alle n.

Avatar von 287 k 🚀

Deine Schlussfolgerung "Subtrahieren ergibt" sieht so aus, als ob Du 2 Ungleichungsketten gliedweise subtrahierst. Das wäre (allgemein) falsch.

Ein anderes Problem?

Stell deine Frage

Willkommen bei der Mathelounge! Stell deine Frage einfach und kostenlos

x
Made by a lovely community